Optimizing Cannon Range on a Vertical Tower

In summary: No. Then something is wrong.If you plug in h=0 do you get the standard result of 45 degrees? No. Then something is wrong.In summary, your equations say that the cannon must be elevated at an angle α to achieve the maximum range. However, if you plug in h=0, you don't get the standard result of 45 degrees. Something is wrong with your steps.
  • #1
jasonchiang97
72
2

Homework Statement



A cannon that is capable of firing a shell at speed v0 is mounted on a vertical tower of height h that overlooks a level plain below.

Show that the elevation angle α at which the cannon must be set to achieve maximum range is given by the expression

csc2(α) = 2(1+gh/V02)

Homework Equations



x(t) = v0tcosα
y(t) = v0tsinα - (1/2)gt2 + h

The Attempt at a Solution



[FONT=PT Sans, san-serif]

First, I used

x(t) = v0tcosα
y(t) = v0tsinα - (1/2)gt2 + h

I solved for t in y(t) and plugged it into x(t) to get

x = [v0cosαsinα+v0cosα(v02sin2α+2gh)1/2]/g

then I solved for v0cosα(v02sin2α+2gh)1/2

and squared both sides to get

(xg - v02cosαsinα)/(v02cos2α) = v02+2gh

I expanded the left side of the equation and simplified with the right hand side to get

(g2/v02)x2 -(2gtanα)x - 2gh = 0

Solved for x to get

x = v02/2(sinα + √(sin2α +2h/v02))

I took the dx/dα = 0 to get the maximum angle

dx/dα = 1+sinα(sin2α +2h/v02)-1/2 = 0

And I said that it is max when sinα = 1 so then

xmax = v02/2 + v02/2(1+2h/v02

I'm unsure on how to obtain cscα from this

[/FONT]
 
Last edited:
Physics news on Phys.org
  • #2
Your equations can't be what you said they are.

jasonchiang97 said:
x(t) = v0tcosα
y(t) = v0tsinα

That says that both the horizontal and vertical motion are at constant velocity. The vertical motion is accelerated, as it is subject to gravity.
jasonchiang97 said:
x = [v0cosαsinα+v0cosα(v02sin2α+2gh)1/2]/g

This looks like, contrary to the equations you said you were using, you used an equation that had a g in it, unlike the above. But I'm having a little trouble figuring out exactly what you're doing.

So what equations did you actually start with and what were your actual steps? Clearly they weren't what you typed here.
 
  • #3
RPinPA said:
Your equations can't be what you said they are.
That says that both the horizontal and vertical motion are at constant velocity. The vertical motion is accelerated, as it is subject to gravity.This looks like, contrary to the equations you said you were using, you used an equation that had a g in it, unlike the above. But I'm having a little trouble figuring out exactly what you're doing.

So what equations did you actually start with and what were your actual steps? Clearly they weren't what you typed here.

Oops I meant to write y(t) = v0tsinα - (1/2)gt2 + y0

where y0 is just h, the height

so y(t) = v0tsinα - (1/2)gt2 + h
 
  • #4
And then what did you do. It looks like you applied the quadratic formula to the equation for ##y(t)## to solve for t in terms of y. Is that so? Why not say so? Why not explain your calculations instead of leaving us to guess if you want us to follow and comment on what you're doing?

The remainder of your calculations seem to be following appropriate steps though I haven't tracked it in detail. Two comments on the final results:
1. ##\csc(\alpha) = 1/\sin(\alpha),## so if you have ##\sin(\alpha)## it's pretty easy to get ##\csc(\alpha)##.
2. You say the maximum range is when ##\sin(\alpha) = 1##, but that's when ##\alpha## is straight up and thus the range is 0. The cannonball lands on your head. Any angle less than that is going to give more range. So clearly there's an error somewhere.

3. There is something else wrong with your final steps. As I just said, if ##\sin(\alpha) = 1##, then the cannon is pointing directly upward. Also ##\cos(\alpha) = 0## which means x(t) = 0 for all t, so I don't know what steps you followed to get a nonzero ##x_{max}## from that. But clearly you're doing something invalid.

I think I see the first place that seems to be going wrong anyway.

jasonchiang97 said:
I took the dx/dα = 0 to get the maximum angle

That's an appropriate thing to do, though x may also have a minimum when ##dx/d\alpha = 0##. So if you find more than one such point, you'll need to check which is which.

jasonchiang97 said:
And I said that it is max when

Why are you looking for where ##dx/d\alpha## is maximized? You just said you wanted to know where ##dx/d\alpha## is 0.
 
Last edited:
  • #5
jasonchiang97 said:
x = [v0cosαsinα+v0cosα(v02sin2α+2gh)1/2]/g
Typo? The expression is dimensionally inconsistent. Need a 2 on the first v0.
jasonchiang97 said:
squared both sides to get

(xg - v02cosαsinα)/(v02cos2α) = v02+2gh
The left hand side is dimensionless, the right is not.
jasonchiang97 said:
dx/dα = 1+sinα(sin2α +2h/v02)-1/2 = 0
Time for a sanity check. If you plug in h=0 do you get the standard result of 45 degrees?
 

1. What is a cannon on a vertical tower?

A cannon on a vertical tower is a type of artillery weapon that is mounted on a tall structure, such as a tower or wall, to provide a higher vantage point for firing projectiles.

2. What is the purpose of a cannon on a vertical tower?

The purpose of a cannon on a vertical tower is to provide a strategic advantage in warfare by allowing for a wider range of firing and increased accuracy due to the elevated position.

3. How does a cannon on a vertical tower work?

A cannon on a vertical tower works by using gunpowder to create a force that propels a projectile, such as a cannonball, out of the barrel at a high speed towards a target.

4. What are the advantages of using a cannon on a vertical tower?

The advantages of using a cannon on a vertical tower include increased range and accuracy, as well as the ability to target and fire at enemies from a protected and elevated position.

5. What are the potential drawbacks of a cannon on a vertical tower?

Potential drawbacks of a cannon on a vertical tower include the need for a sturdy and stable tower structure, as well as the risk of being a visible target for enemies. Additionally, the height of the tower may make it difficult to reload and maintain the cannon.

Similar threads

  • Introductory Physics Homework Help
Replies
34
Views
676
  • Introductory Physics Homework Help
Replies
25
Views
455
  • Introductory Physics Homework Help
Replies
21
Views
2K
Replies
5
Views
366
  • Introductory Physics Homework Help
Replies
32
Views
1K
  • Introductory Physics Homework Help
Replies
6
Views
2K
  • Introductory Physics Homework Help
Replies
2
Views
2K
  • Introductory Physics Homework Help
Replies
4
Views
2K
  • Introductory Physics Homework Help
Replies
4
Views
2K
  • Introductory Physics Homework Help
Replies
18
Views
1K
Back
Top